www.vorkurse.de
Ein Projekt von vorhilfe.de
Die Online-Kurse der Vorhilfe

E-Learning leicht gemacht.
Hallo Gast!einloggen | registrieren ]
Startseite · Mitglieder · Teams · Forum · Wissen · Kurse · Impressum
Forenbaum
^ Forenbaum
Status Mathe-Vorkurse
  Status Organisatorisches
  Status Schule
    Status Wiederholung Algebra
    Status Einführung Analysis
    Status Einführung Analytisc
    Status VK 21: Mathematik 6.
    Status VK 37: Kurvendiskussionen
    Status VK Abivorbereitungen
  Status Universität
    Status Lerngruppe LinAlg
    Status VK 13 Analysis I FH
    Status Algebra 2006
    Status VK 22: Algebra 2007
    Status GruMiHH 06
    Status VK 58: Algebra 1
    Status VK 59: Lineare Algebra
    Status VK 60: Analysis
    Status Wahrscheinlichkeitst

Gezeigt werden alle Foren bis zur Tiefe 2

Navigation
 Startseite...
 Neuerdings beta neu
 Forum...
 vorwissen...
 vorkurse...
 Werkzeuge...
 Nachhilfevermittlung beta...
 Online-Spiele beta
 Suchen
 Verein...
 Impressum
Das Projekt
Server und Internetanbindung werden durch Spenden finanziert.
Organisiert wird das Projekt von unserem Koordinatorenteam.
Hunderte Mitglieder helfen ehrenamtlich in unseren moderierten Foren.
Anbieter der Seite ist der gemeinnützige Verein "Vorhilfe.de e.V.".
Partnerseiten
Weitere Fächer:

Open Source FunktionenplotterFunkyPlot: Kostenloser und quelloffener Funktionenplotter für Linux und andere Betriebssysteme
Forum "Lineare Algebra - Moduln und Vektorräume" - Ergänzung der Basis
Ergänzung der Basis < Moduln/Vektorraum < Lineare Algebra < Hochschule < Mathe < Vorhilfe
Ansicht: [ geschachtelt ] | ^ Forum "Lineare Algebra - Moduln und Vektorräume"  | ^^ Alle Foren  | ^ Forenbaum  | Materialien

Ergänzung der Basis: Frage (beantwortet)
Status: (Frage) beantwortet Status 
Datum: 17:24 Sa 30.10.2010
Autor: kushkush

Aufgabe
3.
Die Vektoren [mm] q=\vektor{1\\-2\\-1} [/mm] und [mm] p=\vektor{2\\1\\1} [/mm] sollen durch die Wahl eines dritten Vektors [mm] d\in \IR^{3} [/mm] zu einer Basis von [mm] \IR^{3} [/mm] ergänzt. Es soll auch gezeigt werden, dass die gewählten Vektoren zusammen auch wirklich eine Basis bilden.

5. Es soll gezeigt werden, dass die drei Vektoren:

[mm] q=\vektor{1+i\\2\\-i} [/mm] und [mm] p=\vektor{i\\-1\\1+i} [/mm] und [mm] b=\vektor{2\\2+i\\1} [/mm] eine Basis des komplexen Vektorraums [mm] \IC [/mm] bilden. Der Vektor [mm] d=\vektor{i+1\\3+4i\\2+2i} [/mm] als Linearkombination dieser Basis.

Hallo!

Zu 1.
Basis-Vektoren müssen linear unabhängig sein. Deshalb wähle ich für b den Vektor, welcher senkrecht auf diese steht:

[mm] $b=\vektor{-4\\1\\5}$ [/mm]

Die andere Eigenschaft ist, dass es möglich ist, durch eine Linearkombination  jeden anderen Vektor in [mm] \IR^{3} [/mm] zu bilden:

[mm] $a\cdot \vektor{-4\\1\\5}+b\cdot \vektor{2\\1\\1}+c\cdot \vektor{1\\-2\\1}=\vektor{a\\b\\c}$ [/mm]

Das scheint aber nur für a,b,c = 0 zu stimmen, also stimmt meine Lösung nicht. Durch welches Verfahren finde ich denn einen 3ten, passenden Basisvektor?

Bei

2.

Also zuerst zeige ich, dass sie lin. unabhängig sind. Dann dass die Linearkombination: [mm] (a+di)\vektor{1+i\\2\\-i}+ (b+ci)\vektor{i\\-1\\1+i}+ (c+oi)\vektor{2\\2+i\\1})=\vektor{a+di\\b+ci\\c+oi} [/mm] ergibt.

Dass der Vektor [mm] \vektor{i+1\\3+4i\\2+2i} [/mm] eine Linearkombination ist, zeige ich durch ein Gleichungssystem.


So weit alles richtig?



Ich habe diese Frage in keinem anderen Forum gestellt und bin für jede Antwort dankbar.

        
Bezug
Ergänzung der Basis: Antwort
Status: (Antwort) fertig Status 
Datum: 17:54 Sa 30.10.2010
Autor: zahllos

Hallo,

zur ersten Aufgabe:

dein Vektor b steht nicht auf den beiden angegebenen Vektoren senkrecht. Da bekommst einen dritten zu den beiden gegebenen Vektoren (ich nenne sie mal u und v) sekrechten Vektor entweder durch das Kreuzprodukt, oder in dem du einen Vektor, der nicht in der von u und v aufgespannten Ebene liegt, auf u und v projizierst. (Wenn dir nicht ganz klar ist wie das geht, sag Bescheid).

Den Nachweis, dass die drei Vektoren linear unabhängig sind, führst du z.B. indem zu zeigst, dass die von ihnen erzeugte Matrix Rang drei hat.



zur zweiten Aufgabe:

hier musst du den Nullvektor als Linearkombination von p,q, und b darstellen und zeigen, dass das nur geht, wenn alle drei Vektoren mit dem Skalar 0 multipliziert werden. Oder du zeigst hier ebenfalls, dass die von diesen drei Vektoren erzeugte Matrix Rang drei hat.


Bezug
                
Bezug
Ergänzung der Basis: Frage (beantwortet)
Status: (Frage) beantwortet Status 
Datum: 18:09 Sa 30.10.2010
Autor: kushkush

Hallo,

Reicht denn für den Nachweis einer Basis nur ein Nachweis der linearen Unabhängigkeit?

Bei 3)
Das Kreuzprodukt ergibt mir den Vektor$  [mm] \vektor{-3\\1\\5}$ [/mm]

Das ergibt mir die Matrix:

[mm] $\vektor{-3 & -1 & 2\\ 1 & -2 & 1 \\ 5 & 1 & 1}$ [/mm]

Die kann ich nicht weiter vereinfachen, also ist der Rang 3 .

Damit wäre ich  fertig?

Bezug
                        
Bezug
Ergänzung der Basis: Antwort
Status: (Antwort) fertig Status 
Datum: 18:45 Sa 30.10.2010
Autor: gollum13

Eine Basis ist ein unverkürzbares Erzeugenden System (du kannst alle anderen Vektoren durch ihre linear Kombination erzeugen, aber wenn du einen Vektor deiner Basis weg lässt geht das nicht mehr). Aus diesem Grund ist auch eine manchmal elegante Lösung um zu zeigen, dass du eine Basis hast, die kanonische Basis aus ihr zu erzeugen (d.h. die Basis wo in jedem Vektor immer nur eine 1 und sonst Nullen stehen)

Wenn du drei unabhängige Vektoren hast, erzeugen dir diese einen Vektorraum der Dimension 3, da dieser ein Unterraum von einem 3-dim. Vektorraum ist, folgt die Gleichheit. Du hast also eine Basis für diesen Vektorraum gefunden und bist fertig.

Bezug
                                
Bezug
Ergänzung der Basis: Frage (beantwortet)
Status: (Frage) beantwortet Status 
Datum: 19:01 Sa 30.10.2010
Autor: kushkush

Also reicht die lineare Unabhängigkeit alleine nicht, sondern ich muss noch 3 Gleichungssysteme aufstellen, wo jeweils jeder einzelne Einheitsvektor rauskommt.

Die Dimension erhalte ich also aus der Anzahl der Basisvektoren.

Ich habe gelesen, dass die Dimension die Anzahl der Vektorkomponenten angibt.

Aber die Anzahl der Vektorkomponenten muss ja nicht zwangsweise gleich der Anzahl der Basisvektoren sein?


Danke

Bezug
                                        
Bezug
Ergänzung der Basis: Antwort
Status: (Antwort) fertig Status 
Datum: 19:08 Sa 30.10.2010
Autor: zahllos

Hallo,

wenn du drei linear unabhängige Vektoren im dreidimensionalen Raum hast, bilden die eine Basis dieses Raums. Wenn die aber drei l.u. Vektoren in einem höherdimensionalen Raums hast, kannst du diese zu einer Basis ergänzen. Bei deiner Aufgabe liegt der erste Fall vor.  

Bist du mit der zweiten Aufgabe weitergekommen?

Bezug
                                                
Bezug
Ergänzung der Basis: Frage (beantwortet)
Status: (Frage) beantwortet Status 
Datum: 19:32 Sa 30.10.2010
Autor: kushkush

Danke!
Zu 1)
Der Vollständigkeit halber muss ich doch jetzt auch noch zeigen, dass 3 l.u. Vektoren im  [mm] $\IR^{3} [/mm] $ auch den ganzen Raum erfassen oder, und das kann ich mit den Einheitsvektoren?
Oder ist das überflüssig weil das immer gilt wenn ich $n$ l.u. Vektoren in einem $n$-dimensionalen Raum habe?
Damit wäre ich bei 1) fertig, weil ich 3 l.u. Vektoren habe und gezeigt habe dass sie auch l.u. sind?

Wie würde ich denn einen l.u. Vektoren finden für einen Vektorraum höherer Dimension als 3?
Zu 2)

Das LGS [mm] $\vektor{1+i & i& 2 \\ 2 & -i & 2+i\\ -i & 1+i & 1}\cdot \vektor{q\\p\\b} [/mm] = [mm] \vektor{0\\0\\0}$ [/mm] lässt sich nicht weiter vereinfachen, also hat es nur die Lösung $q,p$ und $b = 0$ und ist damit l.u.

Zur Unteraufgabe von 2) :

[mm] $\vektor{i+1\\-3+4i\\2+2i}=a\vektor{1+i\\2\\-i}+b\vektor{i\\-1\\1+i}+c\vektor{2\\2+i\\1}$ [/mm]

Das Gleichungssystem hab ich mit Substitution gelöst und erhalten:

$c= [mm] \frac{15}{13}i [/mm] + [mm] \frac{10}{13}$ [/mm]
[mm] $b=\frac{-64i}{65}+\frac{18}{65}$ [/mm]
[mm] $a=\frac{-2i}{65}-\frac{101}{65}$ [/mm]


Bezug
                                                        
Bezug
Ergänzung der Basis: Antwort
Status: (Antwort) fertig Status 
Datum: 19:59 Sa 30.10.2010
Autor: zahllos

Hallo,

bei der ersten Aufgabe bist du fertig. Wenn due einen höherdimensionalen Raum hast, musst du einen Vektor suchen, der nicht von deinen Basisvektoren erzeugt wird und ihn auf die Basisvektoren projizieren.

Bei der zweiten Aufgabe versuche ich mal eine Lösung zu berechnen, aber das kann dauern.


Bezug
                                                                
Bezug
Ergänzung der Basis: Mitteilung
Status: (Mitteilung) Reaktion unnötig Status 
Datum: 21:50 Sa 30.10.2010
Autor: kushkush

Danke!

Bezug
                                                        
Bezug
Ergänzung der Basis: Antwort
Status: (Antwort) fertig Status 
Datum: 20:09 Sa 30.10.2010
Autor: leduart

Hallo
Auch hierwieder:"lässt sich nicht weiter vereinfachen" musst du zeigen, und natürlich kann man es "vereinfachen"
Gruss leduart


Bezug
                                                                
Bezug
Ergänzung der Basis: Mitteilung
Status: (Mitteilung) Reaktion unnötig Status 
Datum: 20:14 Sa 30.10.2010
Autor: kushkush

Ok, aber wenn man die Vereinfachung nicht sieht, aber sieht dass alle drei nicht miteinander kollinear sind, dann reicht das doch auch??

Bezug
                                                        
Bezug
Ergänzung der Basis: Mitteilung
Status: (Mitteilung) Reaktion unnötig Status 
Datum: 21:52 Sa 30.10.2010
Autor: zahllos

Hallo

bei der zweiten Aufgabe könntest du z.B. die Determinante der 3x3-matrix berechnen, sie ist ungleich Null, also sind die drei Vektoren linear unabhängig.

Bei der darstellung des komplexen Vektors durch dir drei Basisvektoren habe ich bei mehreren Versuchen nur Schrott rausbekommen, mein Ansatz war das ganze als Gleichungsystem mit sechs Unbekannten zuschreiben (die Unbekannten sind dann die Realteile bzw. Imaginärteile der Komponenten der Lösung) damit spart man sich das komplexe Rechnen, aber Lösung schaut schlimm aus.

Vielleicht hat jemand anders eine einfachere Idee!


Bezug
                                                                
Bezug
Ergänzung der Basis: Mitteilung
Status: (Mitteilung) Reaktion unnötig Status 
Datum: 21:59 Sa 30.10.2010
Autor: kushkush

Das mit der Determinante ist sehr nützlich!

Danke!!!

Bezug
                        
Bezug
Ergänzung der Basis: Mitteilung
Status: (Mitteilung) Reaktion unnötig Status 
Datum: 20:05 Sa 30.10.2010
Autor: leduart

Hallo
"Kann ich nicht weiter vereinfachen" ist kein Beweis es kann einfach heissen, dass du es nicht kannst.
wenn du die Matrix auf Dreieckform bringst, kann man "sehen" dass die 3 Vektoren lin unabh. sind.
da du den dritten senkrecht auf beiden hast, und die ersten 2 nicht proportional bzw. kolinear sind reicht auch das Argument.
Gruss leduart



Bezug
Ansicht: [ geschachtelt ] | ^ Forum "Lineare Algebra - Moduln und Vektorräume"  | ^^ Alle Foren  | ^ Forenbaum  | Materialien


^ Seitenanfang ^
www.vorkurse.de
[ Startseite | Mitglieder | Teams | Forum | Wissen | Kurse | Impressum ]